What is System: Definition and 1000 Discussions

A system is a group of interacting or interrelated elements that act according to a set of rules to form a unified whole. A system, surrounded and influenced by its environment, is described by its boundaries, structure and purpose and expressed in its functioning. Systems are the subjects of study of systems theory.

View More On Wikipedia.org
  1. Z

    Python Displaying system information using psutil

    Hi, I am getting following error message: I have written the following code which should display the system information of a simple program which I am running as a function: import psutil import osclass Counting_SysInfo: def __init__(self, i): self.i = i def...
  2. M

    Thermodynamics Problem: Cylinder-Piston System with Friction

    Hi guys, First of all I'm sorry for my bad english I'll try to be as clear as possible. I have tried to solve this problem to understand the First Law of Thermodinamics: Q+L=ΔE_t In fact I know L (in the current convention) is the work which the envirorment does on the system but I don't...
  3. P

    Entropy and the Helmholtz Free Energy of a Mass-Piston System

    Attempt at a Solution: Heat Absorbed By The System By the first law of thermodynamics, dU = dQ + dW The system is of fixed volume and therefore mechanically isolated. dW = 0 Therefore dQ = dU The change of energy of the system equals the change of energy of the gas plus the change of energy...
  4. Mina Farag

    Applying the implicit function theorem to a system of equations

    My attempt: According to the implicit function theorem as long as the determinant of the jacobian given by ∂(F,G)/∂(y,z) is not equal to 0, the parametrization is possible. ∂(F,G)/∂(y,z)=4yzMeaning that all points where z and y are not equal to 0 are possible parametrizations. My friend's...
  5. B

    A rotating system of two point particles with inner torque

    Lets say we have a system of two point particles (1. and 2.) which are rotating around an axis. What is written next in my physics course book is: The torque of a 2.body on the 1. body is M21=r1xF21 and the torque of the 1.body on the 2.body is M12=r2xF12. Understandable. But how? There is no...
  6. O

    I How important is our number system for science?

    I know that our laws are as far as the logic goes Universal? Now, can we use other ancient number systems and come up with the same results, or is the success of all the recent science dependent on the efficiency of Hindu Numerals ? Is the logic of all mathematics based on Hindu numerals , and...
  7. jschim

    Oscillating Spring in a Frictionless pulley system

    1. Draw Free body Diagram for each weight. 2. Solve for Tension in Rope. 3. Find Spring Constant. 4. Find omega (w, or angular velocity)
  8. RoloJosh16

    Find the initial acceleration in a system

    Let´ s call ##N_x## the magnitud of the force between the rod and the box and ## N_y## the magnitud of the force between the rod and the surface. ##N_x = ma_c## ##N_x= ma_r## ##mg-N_y=ma_y## The following I think is to find a relation between ##a_r## and ##a_y## and that can be found by...
  9. RoloJosh16

    Mechanics problem in a non-inertial system

    Well, the problem suggests to analyse it in a system attached to the wedge. If mass m ball is slipping upwards then it would have an acceleration along the wedge pointing upwards (in the suggested system). The forces acting on that ball would be the weight, a normal and a force due to the...
  10. A

    Troubleshooting the Partition Function for a System with 3 Spins

    I'm having problems solving the partition function. I've attached a photo of where I currently am. Am I on the right track? What should be my next step?
  11. B

    Time period of a mass spring system

    I have attempted to draw a sketch of this but can't see how the data they gave me help to find time period This is what value I have ended up getting but I believe is wrong Much appreciated for any help!
  12. ZapperZ

    I Ptolemaic Model of the Solar System

    OK, this is more on history of astronomy than anything else. I hope someone has an insight on how they were thinking of things back then. The Ptolemaic model of the Solar system, i.e. the Geocentric model, placed Venus inside the sun's orbit around the earth. And to account for the retrograde...
  13. C

    Thermodynamics help please -- Air passing through a gas turbine system

    Summary:: NO TEMPLATE BECAUSE THIS HOMEWORK PROBLEM WAS MISPLACED IN A REGULAR FORUM Cant do part c, using the steady flow equation I am confused how to continue. Please help! Mainly confused as to what heat transfer loss represents in the steady flow equation and where to go to find the...
  14. Kaushik

    Equilibrium in a system containing I2 and I

    Summary:: . Consider a container consisting of I2 and I in equilibrium. I2 <-> 2I (Is there any to write chemical equations here?) Will both of them separately contribute to the total pressure (as partial pressures)? If no, why? If yes, should we use mole fraction or volume composition to...
  15. J

    Which statements are true given a solution to the system x'=Ax?

    Summary:: Suppose that [x, y] = e^{-3t} [-2, -1] is a solution to the system $x' = Ax$, where A is a matrix with constant entries. Which of the following must be true? a. -3 is an eigenvalue of A. b. [4, 2] is an eigenvector of A. c. The trajectory of this solution in the phase plane with axes...
  16. Pouyan

    Change in Entropy for an isolated system

    ΔU_A + ΔU_B = 0 (Is this because of isolated system am I right?) ΔU_A = CA * (T_final - T_A ) ΔU_B=CB * (T_final-T_B) And because of a very slow process : S=ln(T) T_final= (CA T_A + CB T_B)/(CA + CB) ΔS_final = CA*ln(T_f/TA) + ln(T_f/TB) * CB My QUESTION is : When we say No heat exchange...
  17. Saptarshi Sarkar

    Torque on a rotating system system

    I calculated the total moment of inertia of the system to be ##2ml²Sin²θ##, so the angular momentum is ##2ml²ωSin²θ##. To get the torque on the system I need to differentiate the angular momentum but I don't have any time dependent terms. What should I do?
  18. M

    MHB Determine the solution set of the system using the echelon form

    Hey! :o Let $\displaystyle{a:=\begin{pmatrix}2 & 1 & 0 & 5 \\ 1 & 0 & 1 & 1 \\ 4 & 1 &2 & 7\end{pmatrix}\in \mathbb{R}^{3\times4}}$ and $\displaystyle{b_1:=\begin{pmatrix}1 \\ 1 \\ 1\end{pmatrix} , \ b_2:=\begin{pmatrix}-2 \\ 1 \\ 0\end{pmatrix} \in \mathbb{R}^3}$. I applied the Gauss...
  19. Cetus

    Five State Quantum System, understanding the question

    I’ve never worked with a quantum system with more that two states 1, -1, and I’ve just gotten this homework problem. I'm not sure what it means. Does this mean it has five states? Why are there two 0’s and two 1’s?
  20. BadgerBadger92

    B Time in Cartesian Coordinate Systems: Math Q&A

    I am teaching myself math and have a question about cartesian coordinate systems. How is time illustrated in such a graph? [Moderator's note: Moved from a math forum after post #13.]
  21. N

    Image position in an optical system?

    I have an plano-convex lens with focal length 75 mm. The object is 325mm from the lens. a) Where is the location of the image after the lens? b) Where is the location of the image if an beamsplitter is placed after the lens? My solution, questions: a) a = 325 mm b = ? mm f = 75 mmm...
  22. G

    I Is a bipartite system necessary for the proof of the PBR theorem?

    Hi. I'm trying to grasp what the PBR theorem is about. I'm not tackling the full version, but rather the simple example in @Demystifier's summary. While I think I understand the mathematical steps, my question is why you need two systems to prove it. Is this only technical or more fundamental...
  23. Moises

    I The solar system and the Coriolis effect

    After concluding an investigation about the coriolis effect, I wondered how this phenomenon affected the solar system and in particular the moon. Since the moon is a body that moves within the rotating system formed by the sun and the earth, an apparent deviation in the moon's path would be...
  24. H

    Electronics Portable Solar panel system to run a small computer fan

    Hi all. Recently I purchased two portable USB solar panels, which are rated at 5 watts each. They both came with a 'battery bank' consisting of a single battery all running on 5 volts. I also purchased a 5 volt USB computer fan. My question is; can plug all these things in together so that...
  25. I

    A Building a better ranking system (probability)

    Suppose we have four games and the probability that a player will win the game are as follows: Game 1: 71% Game 2: 55% Game 3: 58% Game 4: 16% Suppose player b won these games with the following percentages of time: Game 1: 100% Game 2: 96% Game 3: 87% Game 4: 67% In other words, he's a very...
  26. Amik

    Velocity of the center of mass of the system

    I believe they are the same?I think i am wrong.
  27. M

    Derivation of the Equations of Motion for a System

    Summary:: This is a system and we want to find the equations of motion. After some force-based attempts, I think that it would be easier to use some energy methods. Hi, I wanted to ask about deriving equations of motion by using the Lagrangian. The question is in the picture below. We are...
  28. L

    Graphing θ=π/4 on a Polar Coordinate System

    When you graph something like ##θ=\frac{π}{4}## on a Polar Coordinate System: Why does the line go into the opposite quadrant as well? I can intuitively understand why it is in the first quadrant: ##θ = 45°## there and so all possible values of ##r## would apply there, giving you a straight line...
  29. R

    System with Pulleys and a Rope That Might Break

    I tried to solve The system of torque as following. I labelled pulley one is the one in the left, and two as the one on the right. α=a/R t1=(-F+T1) R=MR^/2α. So -F+T=Ma/2 for pulley 2 (T1-T2)R=MR^2/2. So T1-T2=Ma/2 for The box T2-mg=ma add the last two equation, to get that T1-mg=Ma/2+ma...
  30. Amik

    Comparing Energy Transfer and Speed Changes in Isolated Systems

    I have difficulty at choosing isolated system.We can not choose rope and block to be the isolated system?This is for number 20.
  31. jisbon

    Finding the distance in an optics system

    Hi all, Pretty new to optics here, so would like to get my basics right. Below is my working, was wondering if my concepts are correct here. First I will separate mirror and lens and solve them 1 by 1, so solving for mirror first, ##\dfrac {1}{p_{1}}+\dfrac {1}{q_{1}}=\dfrac {1}{f_{1}} ##...
  32. currently

    Deriving the first-order system for this governing equation

    I tried finding the solution of the equation itself but it hasn't helped! Links to concepts would be greatly appreciated...thank you...
  33. Kaushik

    What exactly do "state of a system" and "state variables" mean?

    Could someone provide me with the intuitive understanding of state of a thermodynamic system? What does state variables/parameters mean?
  34. Y

    Low-Temp Liquid Helium System: Evaporation Heat & Temperature

    We consider a system composed of liquid helium in equilibrium with its vapor at very low temperature T, each phase being considered extensive. We neglect the mass of the gas compared to that of the liquid, as well as the heat capacities of the gas and the walls compared to that of the liquid...
  35. K

    I Does gravity act differently on a Solar System, a Galasy, and the Universe?

    Since acceleration due to Newton's law decreases as the inverse of distance, it becomes very weak at large distances. Our Sun was unable to pull in matter in the Oort cloud in over 4 billion years. Above about 0.11 light years, using modified Newtonian dynamics equations, gravity decreases by...
  36. LittleRookie

    B Questions about mathematics as a deductive system

    Hello, I have a few questions about mathematics as a deductive system in which mathematical statements follow as a logical consequence from axioms and other statements. In particular, consider the real number system and Euclidean geometry. Is doing arithmetic with numbers and working on...
  37. mastermechanic

    Cost of a 50 km range RC system & Type of Antenna

    Hello everyone, I'm a mechanical engineering student working on a reconnaissance/surveillance UAV system. To summarize it, 1-) It will either have FLIR camera system (If I can afford it) or GoPro with turret system. In either case it will transmit 4K or lower quality video record. It will also...
  38. I

    Bernoulli equation in a closed loop system

    P1 = 5psi P2= 15psi , Z2-Z1 = 0, i assume V2 =V1 because velocity of water is the same everywhere in a pipe of constant diameter is H friction = H pump = 10psi ? Please help
  39. Charlie Cheap

    Automotive Spark length of a points ignition compared to an HEI system

    Having made/modified my own distributors for years, I was surprised to see MSD (Multiple Spark Discharge) company show their graph using Points, HEI, and Performance HEI comparisons, with little difference in Horse Power made, up to about 6000 RPM. I know a points mechanical system makes a...
  40. AL115

    Conservation of momentum in a bullet-block-spring system

    According to the first equation, the final potential energy is equal to the initial kinetic energy of the block. So that means that Vblok is the instantaneous speed of the block right before it moves to the right and compress the spring, right? But doesn't the second equation (The initial total...
  41. hdp12

    Help with a control system for a hydroelectric tank

    Hello there, please help if you are able.. So we have a tank with input volumetric flow and output volumetric flow The system is controlled by the height of water in the tank h(t) = hdes + (√h(t))*(-A2*(√2g) / A1) - h'(t) I am trying to find a transfer function for this system but I do not...
  42. B

    What is the value of the second term in the commutator for an N particle system?

    I have insertet the equations for H and P in the relation for the commutator which gives $$[H,P] = [\sum_{n=1}^N \frac{p_n^2}{2m_n} +\frac{1}{2}\sum_{n,n'}^N V(|x_n-x_n'|),\sum_{n=1}^N p_n] \\ = [\sum_{n=1}^N \frac{p_n^2}{2m_n},\sum_{n=1}^N p_n]+\frac{1}{2}[\sum_{n,n'}^N...
  43. J

    What is the frictional force on a wheel-disk system?

    The figure is in the attached image. My attempt: I(wheel-disk system) = 0.5mr^2 + MR ^2 = 0.5(0.2kg)(0.1m)^2 + (1.5kg)(0.3m)^2 = 0.136(kg)(m^2) Fnet(of object) = Ma Mg - T = Ma T = Mg - Ma Fnet (wheel-disk system) = (M+m)a Mg- Ma - Fr = (M+m)a a = 0 because system in equilibrium Mg -...
  44. Wrichik Basu

    Other How faulty is the current education system?

    Having spent five months as an undergraduate with physics major, I found that there are two types of students who come to study physics: 1. For a job, and 2. For pursuing research. This thread is concerned with the second type only. Among many other problems, here are two that have concerned me...
  45. Kaelor

    Inserting thick lenses into a thin lens system and deducing values

    Homework Statement:: Finding the distance between the back surface of the first lens and the front surface of the back lens. Homework Equations:: 1/f = 1/s_o + 1/s_i I have two positive thin lenses that are separated by a distance of 5 cm. The focal lengths of the lenses are F_1 = 10 cm and...
  46. X

    I Building a solar system simulator

    Hiya Hiya. I'm trying to build a solar system simulator for research purposes, and this does not seem to be all that easy. As in, my firsts tests have the moon shooting out of the solar system... I was wondering if you folks could offer some guidance. So, since it's for research, accuracy is...
  47. M

    Engineering Design a go kart braking system

    i have been stuck on this question from my assignment, i have tried solving it in many ways, but apparently i am either bad at this or i just don't understand the question haha. * As a prudent engineer, the main shaft is now subjected to loading condition when only brake is applied. Show with...
  48. Riotto

    I Single-particle phase spaces for a system of interacting particles

    For a system of interacting particles, is it possible to define single-particle phase spaces? If not, why?
Back
Top